K
Khách

Hãy nhập câu hỏi của bạn vào đây, nếu là tài khoản VIP, bạn sẽ được ưu tiên trả lời.

13 tháng 9 2017

Ta có: S x 3 = 3+1+ 1 3 + 1 9 + 1 27 + 1 81 + 1 243

Vậy S x 3 - S =  3 - 1 729

S x 2 = 2186 729 suy ra S = 2186 729 :2= 1093 729

a: A=2^0+2^1+...+2^9

2A=2+2^2+...+2^10

=>A=2^10-1

b: B=1+3+3^2+...+3^6

=>3B=3+3^2+...+3^7

=>2B=3^7-1

=>\(B=\dfrac{3^7-1}{2}\)

10 tháng 7 2015

S = 1/3+1/9+1/27+1/81+1/243+1/729+1/2187 ( 1 ) 
Nhân S với 3. Ta có: 
S x 3 = 1 + 1/3 + 1/9 + 1/27 + 1/81 + 1/243 + 1/729 ( 2 ) 
Trừ ( 2 ) với ( 1 ) ta có: 
S x 3 - S = 1 - 1/ 2187 
2S = 2186/ 2187 
S = 2186/ 2187 : 2 
S = 1093/ 2187 

11 tháng 9 2016

đặt S=\(\frac{1}{3}+\frac{1}{9}+\frac{1}{27}+\frac{1}{81}+\frac{1}{243}+\frac{1}{729}\)

=>3S= \(1+\frac{1}{3}+\frac{1}{9}+\frac{1}{27}+\frac{1}{81}+\frac{1}{243}\)

=>3S-S=\(\left(1+\frac{1}{3}+...+\frac{1}{243}\right)-\left(\frac{1}{3}+\frac{1}{9}+...+\frac{1}{729}\right)\)

=>s=1-1/729 = 728/729

11 tháng 9 2016

1/3+1/9+1/27+1/81+1/243+1/729=(1/3+1/9+1/81)+(1/27+1/243+1/729)=37/81+37/729=333/729+37/729=370/729

26 tháng 2 2020

\(\frac{1}{3}+\frac{1}{9}+\frac{1}{27}+\frac{1}{81}+\frac{1}{243}\)

\(=\frac{3}{9}+\frac{1}{9}+\frac{1}{27}+\frac{1}{81}+\frac{1}{243}\)

\(=\frac{4}{9}+\frac{1}{27}+\frac{1}{81}+\frac{1}{243}\)

\(=\frac{12}{27}+\frac{1}{27}+\frac{1}{81}+\frac{1}{243}=\frac{13}{27}+\frac{1}{81}+\frac{1}{243}=\frac{39}{81}+\frac{1}{81}+\frac{1}{243}=\frac{40}{81}+\frac{1}{243}\)

\(=\frac{120}{243}+\frac{1}{243}=\frac{121}{243}\)

23 tháng 4 2016

1 + 1/3 + 1/9 + 1/27 + 1/81 + 1/243 + 1/729

=1+ 243/729+ 81/729 + 27/729 + 9/729 + 3/729

=1093/729

23 tháng 4 2016

1 + 1/3 + 1/9 + 1/27 + 1/81 + 1/243 + 1/729

=1093/729

Ta có:\(A=\frac{1}{3}+\frac{1}{9}+\frac{1}{27}+\frac{1}{81}+\frac{1}{243}\)

Xét\(\frac{1}{3}A=\frac{1}{9}+\frac{1}{27}+\frac{1}{81}+\frac{1}{243}+\frac{1}{729}\)

\(\Leftrightarrow A-\frac{1}{3}A=\frac{1}{3}-\frac{1}{729}\)

\(\Leftrightarrow\frac{2}{3}A=\frac{243-1}{729}\Leftrightarrow A=\frac{3}{2}\times\frac{242}{729}=\frac{121}{243}\)

31 tháng 8 2019

Phải là : A=1/3+1/9+1/27+1/81+1/243 ta có: 3A=1+1/3+1/9+1/27+1/81                     3A-A=(1+1/3+1/9+1/27+1/81)-(1/3+1/9+1/27+1/81+1/243)=1-1/243       2A=242/243                                             A=242/243:2=121/243

16 tháng 5 2015

1+ 1 /3+1/9+1/27+1/81+1/243+1/729.
Đặt:
S = 1 + 1/3 + 1/9 + 1/27 + 1/81 + 1/243 
Nhân S với 3 ta có:
S x 3 = 3 +1+ 1/3 + 1/9 + 1/27 + 1/81
Vậy: 
S x 3 - S = 3 - 1/243
2S = 728/243
S = 364/243

tick đúng nha

4 tháng 8 2017

=364/243

13 tháng 9 2018

Đặt \(A=\frac{1}{3}+\frac{1}{9}+\frac{1}{27}+\frac{1}{81}+\frac{1}{243}\) ta có : 

\(A=\frac{1}{3}+\frac{1}{3^2}+\frac{1}{3^3}+\frac{1}{3^4}+\frac{1}{3^5}\)

\(3A=1+\frac{1}{3}+\frac{1}{3^2}+\frac{1}{3^3}+\frac{1}{3^4}\)

\(3A-A=\left(1+\frac{1}{3}+\frac{1}{3^2}+\frac{1}{3^3}+\frac{1}{3^4}\right)-\left(\frac{1}{3}+\frac{1}{3^2}+\frac{1}{3^3}+\frac{1}{3^4}+\frac{1}{3^5}\right)\)

\(2A=1-\frac{1}{3^5}\)

\(A=\frac{1-\frac{1}{3^5}}{2}\)

Vậy \(A=\frac{1-\frac{1}{3^5}}{2}\)

Chúc bạn học tốt ~ 

6 tháng 11 2016

https://www.youtube.com/watch?v=fvGaHwKrbUc

6 tháng 11 2016

Đặt A = 1/3 + 1/9 + 1/27 + 1/81 + 1/243 + 1/729 
A x 3 = 3 x (1/3 + 1/9 + 1/27 + 1/81 + 1/243 + 1/729) 
= 1 + 1/3 + 1/9 + 1/27 + 1/81 + 1/243 
A x 3 - A = 1 + 1/3 + 1/9 + 1/27 + 1/81 + 1/243 - (1/3 + 1/9 + 1/27 + 1/81 + 1/243 + 1/729) 
= 1 + 1/3 + 1/9 + 1/27 + 1/81 + 1/243 - 1/3 - 1/9 - 1/27 - 1/81 - 1/243 - 1/729 
= 1 - 1/729 
A x 2 = 728/729 
A = 364/729

2 tháng 8 2015

đặt biểu thức trên là A

ta có : 

A= ghi biểu thức ra

A.3=3.(1+1/3+1/9+1/27+1/81+1/243+1/729)

A.3=3+1+1/3+1/9+1/27+1/81+1/243

A.3-A=...

A.2=3-1/729

sau đó bn tự tính ra